Difference between revisions of "1996 AIME Problems/Problem 11"

(See also)
(Problem)
Line 1: Line 1:
 
== Problem ==
 
== Problem ==
{{empty}}
+
Let <math>\mathrm {P}</math> be the product of the roots of <math>z^6+z^4+z^3+z^2+1=0</math> that have an imaginary part, and suppose that <math>\mathrm {P}=r(\cos{\theta^{\circ}}+i\sin{\theta^{\circ}})</math>, where <math>0<r</math> and <math>0\leq \theta <360</math>. Find <math>\theta</math>.
 +
 
 
== Solution ==
 
== Solution ==
 
{{solution}}
 
{{solution}}

Revision as of 16:08, 24 September 2007

Problem

Let $\mathrm {P}$ be the product of the roots of $z^6+z^4+z^3+z^2+1=0$ that have an imaginary part, and suppose that $\mathrm {P}=r(\cos{\theta^{\circ}}+i\sin{\theta^{\circ}})$, where $0<r$ and $0\leq \theta <360$. Find $\theta$.

Solution

This problem needs a solution. If you have a solution for it, please help us out by adding it.

See also

1996 AIME (ProblemsAnswer KeyResources)
Preceded by
Problem 10
Followed by
Problem 12
1 2 3 4 5 6 7 8 9 10 11 12 13 14 15
All AIME Problems and Solutions